Rotations in Bloch Sphere about an arbitrary axis

Click For Summary
SUMMARY

The discussion focuses on the mathematical representation of rotations in the Bloch sphere for a single qubit, specifically the rotation operator \( R_{\hat{n}}(\theta) \) about an arbitrary axis \( \hat{n} \). Participants clarify that the rotation matrices \( R_x(\theta) \), \( R_y(\theta) \), and \( R_z(\theta) \) can be expressed in terms of Pauli matrices, and derive \( R_{\hat{n}}(\theta) = \cos(\frac{\theta}{2})I - i \sin(\frac{\theta}{2})(n_x\sigma_x+n_y\sigma_y+n_z\sigma_z) \). The challenge lies in proving that \( R_{\hat{n}}(\theta) \) indeed represents a rotation about the \( \hat{n} \) axis by the angle \( \theta \), with suggestions for a proof by demonstration involving the properties of Bloch vectors.

PREREQUISITES
  • Understanding of Bloch sphere representation of qubits
  • Familiarity with rotation operators in quantum mechanics
  • Knowledge of Pauli matrices and their applications
  • Basic concepts of vector projections and dot products
NEXT STEPS
  • Study the derivation of rotation operators \( R_x(\theta) \), \( R_y(\theta) \), and \( R_z(\theta) \) using Pauli matrices
  • Explore the mathematical proof of \( R_{\hat{n}}(\theta) \) as a rotation operator
  • Investigate the geometric interpretation of Bloch vectors and their transformations
  • Learn about the implications of rotations in quantum computing and quantum state manipulation
USEFUL FOR

Quantum physicists, quantum computing researchers, and students studying quantum mechanics who seek to understand the mathematical foundations of qubit rotations in the Bloch sphere.

polyChron
Messages
2
Reaction score
0
Hey,
(I have already asked the question at http://physics.stackexchange.com/questions/244586/bloch-sphere-interpretation-of-rotations, I am not sure this forum's etiquette allows that!)

I am trying to understand the following statement. "Suppose a single qubit has a state represented by the Bloch vector ##\vec{\lambda}##. Then the effect of the rotation ##R_{\hat{n}}(\theta)## on the state is to rotate it by an angle $\theta$ about the ##\hat{n}## axis of the Bloch sphere. This fact explains the rather mysterious looking factor of two in the definition of the rotation matrices."
I could work out that the rotation operators ##R_x(\theta)##, ##R_y(\theta)## and ##R_z(\theta)## are infact rotations about the ##X,Y## and ##Z## axis. But how do I extend this for ##R_{\hat{n}}(\theta)## and prove the above statement. Please point me in the right direction.

Thanks.
 
Physics news on Phys.org
I think what you want is what the pauli spin observable is along the "\hat{n}" axis.
Where \hat{n} is a unit vector with components (n_{x},n_{y},n_{z}),

\sigma_{\hat{n}} = \sigma_{x}n_{x} + \sigma_{y} n_{y} + \sigma_{z} n_{z}.

If you know what R_{x}(\theta) is in terms of \sigma_{x}, then I think you can work out what R_{\hat{n}}(\theta) is in terms of \sigma_{\hat{n}}.
 
jfizzix said:
I think what you want is what the pauli spin observable is along the "\hat{n}" axis.
Where \hat{n} is a unit vector with components (n_{x},n_{y},n_{z}),

\sigma_{\hat{n}} = \sigma_{x}n_{x} + \sigma_{y} n_{y} + \sigma_{z} n_{z}.

If you know what R_{x}(\theta) is in terms of \sigma_{x}, then I think you can work out what R_{\hat{n}}(\theta) is in terms of \sigma_{\hat{n}}.

Thanks for your reply.
Yes I have figured that out as well. Let me explain.
I know the rotation matrices in terms of the Pauli matrices, i.e R_x(\theta) = e^{-i\sigma_x /2} and the rotation matrices for \sigma_y and \sigma_z follows in the same manner. I could also prove that R_{\hat{n}}(\theta) = cos(\frac{\theta}{2})I - i sin(\frac{\theta}{2})(n_x\sigma_x+n_y\sigma_y+n_z\sigma_z) using the Taylor expansion. But the difficulties for me start from here. How do I show that R_{\hat{n}}(\theta) is infact a rotation about \hat{n} axis by \theta. How can I construct a concrete proof?
 
polyChron said:
Thanks for your reply.
Yes I have figured that out as well. Let me explain.
I know the rotation matrices in terms of the Pauli matrices, i.e R_x(\theta) = e^{-i\sigma_x /2} and the rotation matrices for \sigma_y and \sigma_z follows in the same manner. I could also prove that R_{\hat{n}}(\theta) = cos(\frac{\theta}{2})I - i sin(\frac{\theta}{2})(n_x\sigma_x+n_y\sigma_y+n_z\sigma_z) using the Taylor expansion. But the difficulties for me start from here. How do I show that R_{\hat{n}}(\theta) is infact a rotation about \hat{n} axis by \theta. How can I construct a concrete proof?

I think what you should do is a proof by demonstration. Compare a bloch vector before \hat{u} and after \hat{u}' a rotation about the n-axis.
Both \hat{u} and \hat{u}' dotted with \hat{n} should give the same value, and the vectors themselves should have the same magnitude. This proves that R_{n}(\theta) is at least some sort of rotation about the \hat{n}-axis.

To find the angle, you need to project \hat{u} and \hat{u}' onto the plane perpendicular to \hat{n}. The dot product of these projected vectors will be the magnitude of each times the cosine of the angle between them, and hopefully that angle will be none other than \theta.
 

Similar threads

  • · Replies 1 ·
Replies
1
Views
2K
  • · Replies 4 ·
Replies
4
Views
3K
  • · Replies 4 ·
Replies
4
Views
2K
  • · Replies 4 ·
Replies
4
Views
1K
Replies
1
Views
2K
  • · Replies 6 ·
Replies
6
Views
3K
  • · Replies 10 ·
Replies
10
Views
18K
  • · Replies 2 ·
Replies
2
Views
1K
Replies
63
Views
8K
  • · Replies 2 ·
Replies
2
Views
3K